0
$\begingroup$

I am struggling with 1.11 exercise from the George Shakan "Discrete Fourier Transform".


Let $A \subset \mathbb{Z}/q\mathbb{Z}$ be any set not containing zero with $|A|>\sqrt2q^{5/8}$. Show that: $$(A+A).(A+A)+A.A+A.A = \mathbb{Z}/q\mathbb{Z}$$ with $$A+B=\{a+b: a\in A, b \in B\}$$ and $$A.B=\{ab: a\in A, b \in B\}$$


My main idea is prove that
$$|(A+A).(A+A)+A.A+A.A| \geq q$$ I tried with Garaev's Theorem:

Let $A \subset \mathbb{Z}/q\mathbb{Z}$ be any nonempty set not containing zero. Then $$\text{max}\{|A+A|,|A.A|\}\geq\text{min}\{\frac{\sqrt{q|A|}}{\sqrt2},\frac{|A|^2}{2\sqrt{q}}\}$$

So I'm thinking about two cases: $|A+A| = \text{max}\{|A+A|,|A.A|\}$ and $|A.A| = \text{max}\{|A+A|,|A.A|\}$
With each, I estimate one part of the left hand side.
For example, with $|A+A| = \text{max}\{|A+A|,|A.A|\}$, I will estimate $|(A+A).(A+A)|$ and try to prove that $|(A+A).(A+A)| \geq q$
Using Garaev's, in this case I have $$|A+A|\geq\text{min}\{\frac{\sqrt{q|A|}}{\sqrt2},\frac{|A|^2}{2\sqrt{q}}\}$$ Since $|A|>\sqrt2q^{5/8}$ as condition, then $$|A+A|\geq\text{min}\{\frac{q^{13/16}}{2^{1/4}},q^{3/4}\}$$ With $q \geq 16, q^{3/4}$ is smaller, so I use it as min here, because the other case is finite.
But then, using the same techniques as the author, I can only prove that $|(A+A).(A+A)| \geq \frac{q}{2}$, which is not large enough and I can't estimate the rest part, too.
I'm wondering if my way of proving this is true or not. Hopefully, someone can help. Thanks in advance!


I wrote down my proof here:
https://drive.google.com/file/d/1cnHfJDlO3eYdskxoFb_sLytlhZSsybQg/view?usp=sharing
And this is George Shakan's paper:
https://gshakan.files.wordpress.com/2016/04/discretefouriertransform1.pdf

$\endgroup$
6
  • $\begingroup$ have you tried this? let $ B = A+A, C = A.A$ and set $b:= |B|,c:= |C|$. by 1.10 we have $bc \ge \min\{q|A|/2, |A|^4/4q\}$. now let $f(b)$ be a lower bound for $|B.B|$ when $|B|\ge b$. similarly let $g(c)$ be a lower bound for $|C+C|$ when $|C|\ge c$. by Cauchy-Davenport it suffices to show $f(b)+g(c)\ge q$ (given the lower bound on $bc$). $\endgroup$ May 28, 2022 at 19:39
  • $\begingroup$ actually looking at your argument, $q/2$ seems to be a rather weak lower bound given the inequality. shouldn’t you be able to deduce $|(A+A).(A+A)|\ge q-q^{3/4}$? $\endgroup$ May 28, 2022 at 19:44
  • 1
    $\begingroup$ @ZachHunter Thanks! I didn't think about using Cauchy-Davenport to compare $|B.B + C + C|$ with $|B.B| + |C+C|$, so I had to split it into two cases. I will try this way, hope it will work. $\endgroup$
    – Sei
    May 30, 2022 at 3:58
  • $\begingroup$ @ZachHunter i'm wondering which part of my argument make the result be a weak lower bound? Is there any better set of solutions S while double-counting is unnecessary? $\endgroup$
    – Sei
    Jun 3, 2022 at 6:47
  • $\begingroup$ literally just looking at the final two lines. if you have that $|B.B| + q^{1/4}\sqrt{|B.B|} -q > 0$, then $|B.B| > q- q^{3/4}$ (since $|sqrt{|B.B|}\le q$ always holds). and this is considerably better than just saying it has size $q/2$. $\endgroup$ Jun 3, 2022 at 6:56

0

Your Answer

By clicking “Post Your Answer”, you agree to our terms of service and acknowledge that you have read and understand our privacy policy and code of conduct.